Đến nội dung

moonkey01

moonkey01

Đăng ký: 22-05-2016
Offline Đăng nhập: 17-09-2018 - 20:07
****-

#657051 $\boxed{\text{Đề thi chọn đội tuyển THPT Chuyên ĐH V...

Gửi bởi moonkey01 trong 07-10-2016 - 21:59

Ý 7.1 thì $a=2016$ nhé.
Còn ý 7.2 thì chứng minh $b$ chia hết cho 6.
Xét $d>6$ với mod 7 thấy vô lý.
Xét d=6 thì tìm đc $b=6,c=4$

 

Bạn đúng rồi, tại mình quên cộng 1 đơn vị nên nhầm :) Cảm ơn bạn :) 




#657045 $\boxed{\text{Đề thi chọn đội tuyển THPT Chuyên ĐH V...

Gửi bởi moonkey01 trong 07-10-2016 - 21:38

Bài 7 dài quá, có thể phân ra 2 ý như sau:

 

7.1/ Tìm tất cả các số $a$ thoả mãn đề bài.

 

Khai triển và thu gọn, ta có $p(2p+3)=q(q^2-q-1)$. Do $(q,q^2-q-1)=1$ và $p$ là số nguyên tố nên chỉ xảy ra $p|q$ hoặc $p|q^2-q-1$.

 

Nếu $p|q$ thì do $p$, $q$ là số nguyên tố nên $p=q$. Thay vào tính được $p=q=4$ hoặc $p=q=-1$, đều loại.

 

Nếu $p|q^2-q-1$ thì $q|2p+3$. Đặt $2p+3=kq(k\in\mathbb{N},k\geq 1)$, thay vào đẳng thức ban đầu, khai triển và thu gọn thì $2q^2-(2+k^2)q+3k-2=0$. Xem đây là phương trình bậc hai theo $q$, để có nghiệm nguyên thì cần có $\bigtriangleup$ là số chính phương. Tính được $\bigtriangleup =k^4+4k^2-24k+20$. Thử với $k$ từ $1$ đến $5$, ta chọn $k\in \left \{ 1,2,5 \right \}$. Thế vào tính lại, nhận $k=5$, $q=13$, $p=31$, $a=2015$. Với $k\geq 6$ thì $(k^2+2)^{2}>k^4+4k^2-24k+20>(k^2)^{2}$ nên $k^4+4k^2-24k+20=(k^2+1)^{2}$, Phương trình này không cho nghiệm nguyên nên loại. Vậy $a=2015$.

 

7.2/ Tìm tất cả các số $b,c,d$ thoả mãn đề bài.

 

Từ giả thiết, $3d!+1>2015$ nên suy ra $d\geq 2$ hay $2|d!$. Ta có đẳng thức $3d!+1-2015-2^{b}=3^c$. Vế trái là bội của 2 mà vế phải thì không, mâu thuẫn. Vậy không tồn tại bộ 4 số nguyên dương thoả mãn đề bài.




#656685 Đề thi chọn đội tuyển Quốc gia tỉnh Vũng Tàu năm 2016-2017

Gửi bởi moonkey01 trong 04-10-2016 - 18:39

Bài 1a:

 

Với $x,y,z$ là các số thực dương, theo bất đẳng thức AM-GM, ta có:

 

$\sum \frac{1}{(2x+y+z)^2}=\sum \frac{1}{((x+y)+(x+z))^2}\leq \sum \frac{1}{4(x+y)(x+z)}$

 

Do đó ta cần chứng minh rằng $\sum \frac{1}{4(x+y)(x+z)}\leq \frac{3}{16}$ hay $\frac{8}{3}(x+y+z)\leq (x+y)(y+z)(z+x)$

 

Theo một kết quả quen thuộc và AM-GM, $(x+y)(y+z)(z+x)\geq \frac{8}{9}(a+b+c)(ab+bc+ca)\geq \frac{8}{9}(a+b+c)\sqrt[3]{a^2b^2c^2}=\frac{8}{3}(x+y+z)$.

 

Từ đó ta có điều phải chứng minh.




#656683 Đề thi chọn đội tuyển Quốc gia tỉnh Vũng Tàu năm 2016-2017

Gửi bởi moonkey01 trong 04-10-2016 - 18:24

Bài 4a:

 

Vế phải của phương trình ban đầu là một hàm bậc nhất theo $y$ nên $f$ là toàn ánh.

 

Lại thay $x$ bởi $f(x)$ vào phương trình ban đầu, dễ chứng minh được $f$ là đơn ánh, hay $f$ là song ánh.

 

Do đó $\exists! a$ sao cho $f(a)=0$. Thay $y$ bởi $a$ vào phương trình ban đầu, ta có $f(x)=a-2017f(x)$ hay $f(x)=c$ ($c$ là hằng số thực).

 

Thử lại ta thấy không thoả mãn, vậy không tồn tại hàm số thoả mãn yêu cầu đề bài.




#656660 Chứng minh $ A, X, Y, Z$ đồng viên.

Gửi bởi moonkey01 trong 04-10-2016 - 13:18

Xét vị trí các điểm như hình vẽ, các trường hợp khác chứng minh tương tự.

 

Gọi $E,F$ là hình chiếu của $H$ trên $CA,AB$ thì $E,F\in (AH)$. Ta có $\angle AFZ=\angle AEZ$, cùng với $\angle ABZ=\angle ACZ$ nên $\angle BZF=\angle CZE$ hay $\bigtriangleup BZF\sim \bigtriangleup CZE(g.g)$ . Từ đó có tỉ lệ $\frac{ZB}{ZC}=\frac{BF}{CE}=\frac{HF}{HE}=\frac{FY}{EX}=\frac{BY}{CX}$ và $\bigtriangleup BZY\sim \bigtriangleup CZX(c.g.c)$, dẫn đến $\angle BZY=\angle CZX$. Có biến đổi góc $\angle XZY=\angle CZY+\angle CZX=\angle CZY+\angle BZY=\angle BZC=\angle BAC$. Vậy $A,X,Y,Z$ cùng thuộc một đường tròn.

 

Hình vẽ gửi kèm:

 

 

Hình gửi kèm

  • Capture.JPG



#656581 Tuần 1 tháng 10/2016: Tiếp tục với vấn đề vuông góc

Gửi bởi moonkey01 trong 03-10-2016 - 18:52

Em có một hướng xử lý khác, có thể dài hơn:

 

Xét vị trí các điểm như hình vẽ, các trường hợp còn lại chứng minh tương tự.
 
Ta có $\angle GEC=\angle GAC=\angle GBP$ nên $BPGE$ nội tiếp. Tương tự có $CPGF$ nội tiếp nên $\angle PGF+\angle PGE=\angle ACP+\angle ABP=180^{o}$ hay $E,F,G$ thẳng hàng. Lại có $\angle GDP=\angle ACE=\angle AGE$ nên $DP\parallel EF$, dẫn đến $\angle GPD=\angle PGF=\angle ACE=\angle GDP$ hay tam giác $GDP$ cân tại $G$, $OG\perp DP\parallel EF$. Gọi $AP$ cắt $BC$ tại $T$ thì theo định lý $Brokard$ với $(O)$, $OT\perp EF$ nên $\overline{O,T,G}$ và tam giác $TDP$ cân tại $T$. Có $\angle BAP=\angle CAQ$ nên $PQ\parallel BC$, từ đó $\angle DAP=\angle DQP=\angle DRT$ nên $ATDR$ nội tiếp. Do đó $\angle RAQ=\angle RAP+\angle PAQ=\angle TDQ+\angle PDQ=\angle TDP=\angle TPD=\angle RQA$ nên tam giác $RAQ$ cân tại $A$. Vậy $OR\perp AQ$.
 
Hình vẽ gửi kèm:

Ảnh chụp Màn hình 2016-10-03 lúc 18.38.15.png




#649413 ​$f(x-f(y))=3f(x)-2x-f(y)$

Gửi bởi moonkey01 trong 13-08-2016 - 17:12

Tìm tất cả các hàm $f:\mathbb{R}\rightarrow \mathbb{R}$ thoả mãn điều kiện:

 

$f(x-f(y))=3f(x)-2x-f(y) \forall x,y\in \mathbb{R}$




#647753 Chọn đội tuyển HSG quốc gia 2015-2016 PTNK( ngày 1)

Gửi bởi moonkey01 trong 03-08-2016 - 13:30

Lời giải cho bài 4, ngày 2:

a) Gọi $V$ là giao điểm của $(AEF)$ với $(O)$. Vẽ đường kính $AL$ của $(O)$ cắt $(AEF)$ tại $S$ khác $A \Rightarrow$ $A$, $S$, $T$ thẳng hàng. Qua $T$ lấy $I'\in VL$ sao cho $TI'//AL$. Ta có $\widehat{OAC}+\widehat{AFE}=90^{o}-\frac{\widehat{BOC}}{2}+\widehat{ABC}=90^{o}$ nên $OA\bot EF$. Nên ta có $I'$ nằm trên trung trực $EF$. Mặt khác, do $OI'$ là đường trung bình của $\Delta ASL$ nên $I'$ cũng nằm trên trung trực $BC\Rightarrow I'\equiv I$. Do đó $I\in VL$. 

$VL$ lần lượt cắt $EF$ và $BC$ tại $X$ và $Y$. Bằng việc sử dụng trục đẳng phương, ta có $AV$, $EF$, $BC$ đồng qui (gọi điểm đó là $U$). Từ đó, ta cũng có được $X$ là trực tâm $\Delta AUL$ nên $AX$ cắt $UL$ tại một điểm thuộc đường tròn $(O)$ (gọi điểm này là $Z$).

Ta có $(FEXU)=(EFUX)=A(BCVZ)=L(BCVZ)=(BCYU)$ nên $BF$, $CE$, $XY$ đồng qui tại D nên ta có $đpcm$.

 

 

Câu a có thể làm ngắn gọn hơn: 

 

Gọi $V$ là giao điểm thứ hai của $(AEF)$ với $(O)$ như trên thì dễ chỉ ra $AV$, $EF$, $BC$ đồng quy. Theo định lý Brokard thì $ID \perp AV$, lại có $OT \perp AV$ do $AV$ là dây chung của $(O)$ và $(T)$ nên $OT \parallel ID$.




#646069 Inequalities From 2016 Mathematical Olympiads

Gửi bởi moonkey01 trong 22-07-2016 - 22:06

Bài 51.1. (Tuymaada, Junior). Cho ba số thực không âm $a,b,c$ thỏa mãn $a^2+b^2+c^2 \geqslant 3.$ Chứng minh rằng
\[(a+b+c)^3 \geqslant 9(ab+bc+ca).\]

 

Giải: Áp dụng bất đẳng thức AM-GM cho 3 số dương, ta có:

 

$(a+b+c)^3+(a+b+c)^3+27\geq 9(a+b+c)^2$

 

$\Rightarrow (a+b+c)^3 +\frac{27}{2}\geq \frac{9}{2}(a+b+c)^2=\frac{9}{2}(a^2+b^2+c^2)+9(ab+bc+ca)\geq \frac{27}{2}+9(ab+bc+ca)$

 

Từ đó ta có điều phải chứng minh. ĐTXR $\Leftrightarrow a=b=c=1$




#645573 Inequalities From 2016 Mathematical Olympiads

Gửi bởi moonkey01 trong 19-07-2016 - 19:27

Bài 50 (Taiwan TST Round 2). Cho hai số thực dương $x,y$ thỏa mãn điều kiện $x+y=1.$ Chứng minh rằng
\[\frac{x}{x^2+y^3}+\frac{y}{x^3+y^2} \leqslant 2 \left(\frac{x}{x+y^2}+\frac{y}{x^2+y}\right).\]
 

 

Giải: Áp dụng bất đẳng thức Cauchy, ta có $(x^2+y^3)(1+y)\geq (x+y^2)^2>0$. Suy ra $\frac{x}{x^2+y^3}\leq \frac{x(1+y)}{(x+y^2)^2}$.

 

Tương tự ta có $\frac{y}{y^2+x^3}\leq \frac{y(1+x)}{(y+x^2)^2}$. Cộng vế theo vế với BĐT trên, ta có:

 

$\frac{x}{x^2+y^3}+\frac{y}{y^2+x^3}\leq \frac{x(1+y)}{(x+y^2)^2}+\frac{y(1+x)}{(y+x^2)^2}$.

 

Ta sẽ chứng minh $\frac{x(1+y)}{(x+y^2)^2}+\frac{y(1+x)}{(y+x^2)^2} \leq 2\left (\frac{x}{x+y^2}+\frac{y}{y+x^2} \right ) (1)$

 

Ta có:

 

$\frac{x(1+y)}{(x+y^2)^2}-\frac{2x}{x+y^2}=\frac{x}{x+y^2}\left ( \frac{1+y}{x+y^2}-2 \right )=\frac{x(1+y-2y^2-2x)}{(x+y^2)^2}=\frac{-x^2(x-y)}{(x+y^2)^2}$

 

$\frac{y(1+x)}{(y+x^2)^2}-\frac{2y}{y+x^2}=\frac{-y^2(y-x)}{(y+x^2)^2}$

 

$(1)\Leftrightarrow \frac{-x^2(x-y)}{(x+y^2)^2}+\frac{-y^2(y-x)}{(y+x^2)^2}\leq 0 \Leftrightarrow -(x-y)\left ( \frac{x^2}{(x+y^2)^2}-\frac{y^2}{(y+x^2)^2} \right )\leq 0\Leftrightarrow \frac{-(x-y)^2(x^2+xy+y^2)}{(x+y^2)(y+x^2)}\left ( \frac{x}{x+y^2}+\frac{y}{y+x^2} \right )\leq 0$ (đúng)

 

Từ đó ta có điều phải chứng minh. ĐTXR $\Leftrightarrow x=y=\frac{1}{2}$




#644861 Inequalities From 2016 Mathematical Olympiads

Gửi bởi moonkey01 trong 13-07-2016 - 22:13

Bài 48: Ta xét: $S=(a^2+ac+c^2)(b^2+bd+d^2)-(ab+bc+cd)^2=(ad-bc)(ab+ad+cd)=(ad-bc)(1+ad-bc)=(ad-bc+\frac{1}{2})^2-\frac{1}{4}\geq -\frac{1}{4}$

$\Rightarrow P\geq (ab+bc+cd)^2-\frac{1}{4}=\frac{3}{4}$

Đẳng thức xảy ra tương đương với việc ta có hệ: $\left\{\begin{matrix} ad-bc=-\frac{1}{2}\\ ab+bc+cd=1\\ \end{matrix}\right.$

Hệ này có nghiệm, ví dụ như bộ nghiệm: $(a,b,c,d)\sim (0,\frac{1}{2},1,\frac{1}{2})$

Như vậy ta có giá trị nhỏ nhất của $P$ là $\frac{3}{4}$, xảy ra chẳng hạn khi $(a,b,c,d)\sim (0,\frac{1}{2},1,\frac{1}{2})$.

 

Làm thế nào để có cách tách "kỳ diệu" như thế nhỉ :D




#644641 Thảo luận về Đề thi và Lời giải của IMO 2016

Gửi bởi moonkey01 trong 12-07-2016 - 13:52

Bài 5: Đẳng thức $(x-1)(x-2)...(x-2016)=(x-1)(x-2)...(x-2016)$ được viết trên bảng với 2016 thừa số ở mỗi vế. Xác định giá trị nhỏ nhất có thể của $k$ để ta có thể xoá $k$ trong số 4032 thừa số trên, với điều kiện là mỗi vế còn lại ít nhất một thừa số, để phương trình nhận được sau phép biến đổi là vô nghiệm.

 

Giải: Ta sẽ chứng minh rằng $k=2016$ là giá trị nhỏ nhất cần tìm.

 

Nếu như ta xoá ít hơn 2016 thừa số, thì đẳng thức nhận được chứa ít nhất 2017 thừa số. Do có 2016 loại thừa số là $x-1$, $x-2$,..., $x-2016$ nên theo nguyên lý Dirichlet, tồn tại một thừa số xuất hiện 2 lần. Thừa số ấy xuất hiện ở cả 2 vế của đẳng thức, dẫn đến việc phương trình nhận được sẽ có nghiệm. Do đó $k\geqslant 2016$

 

Xét phương trình $(x-1)(x-3)...(x-2015)=(x-2)(x-4)...(x-2016)$ $(1)$ nhận được sau khi xoá $k=2016$ thừa số từ đẳng thức ban đầu.

 

Trường hợp $x\in \left \{ 1;2;...;2016 \right \}$ thì phương trình hiển nhiên vô nghiệm.

 

Với $x< 1$ thì $2015-x<2016-x$ ,..., $1-x<2-x$. Cả 2 vế của các bất đẳng thức trên đều dương nên nhân vế theo vế, $VT(1)<VP(1)$. Phương trình vô nghiệm.

 

Với $1<x<2$ thì $VT(1)<0<VP(1)$ nên phương trình cũng vô nghiệm.

 

Với $2<x<3$ thì $2015-x<2016-x$ ,..., $3-x<4-x$, $x-1<x-2$. Cả 2 vế các BĐT trên đều dương nên nhân vế theo vế và đổi dấu, $VT(1)>VP(1)$. Phương trình vô nghiệm.

 

Với $3<x<4$ thì $VT(1)>0>VP(1)$ nên phương trình cũng vô nghiệm.

 

Các trường hợp còn lại ta chứng minh tương tự.

 

Kết luận: Giá trị nhỏ nhất của k là 2016.